All Exams  >   GMAT  >   35 Days Preparation for GMAT  >   All Questions

All questions of Daily Practice Tests for GMAT Exam

Scientists recently documented that influenza spreads around the world more efficiently in the modern era due to commercial air travel. Symptoms of a pandemic-level flu are severe enough that the ill would likely cancel or reschedule air travel, but an infected person can travel across the globe before the first signs appear. Further, if symptoms develop while someone is still on a plane, the infected person's cough can spread the virus easily in the enclosed and closely packed environment.
Which of the following would best minimize the role air travel can play in the spread of influenza during a pandemic?
  • a)
    installing air filtration systems in the planes to kill any flu virus particles flowing through the filters
  • b)
    requiring air travelers to receive flu vaccinations far enough in advance of the trip to provide protection against the disease
  • c)
    refusing to allow children, the elderly, or others who are especially vulnerable to flu to travel by air during a pandemic
  • d)
    requiring all air travelers to wash their hands before boarding a plane
  • e)
    conducting medical examinations during the boarding process to weed out passengers with flu symptoms
Correct answer is option 'B'. Can you explain this answer?

(A) The passage states that the infection can be spread by coughing. The flu virus, therefore, can reach the other
passengers in the “closely-packed environment” before it enters any filters that might kill the virus.
(B) CORRECT. Vaccines provide significant protection against developing the virus (not 100% protection, but you
are asked to “minimize” the impact of air travel, not eliminate it entirely). If all passengers are vaccinated against the
virus, many of those who otherwise would have developed the disease will not, and, therefore, won’t spread it to
others.
(C) Anyone can contract the virus and subsequently spread it; the mentioned populations are merely “especially
vulnerable” to it. Infected people traveling to another place can infect children, senior citizens, and others who have
stayed in their home regions.
(D) The passage states that the infection can be spread by coughing; while it may be true that the virus can also spread
via hand contact, this information is not stated in the passage.
(E) The passage states that people who develop symptoms before travel begins likely would not make the trip; weeding
out those with observable symptoms, then, won’t “minimize” the role of air travel because there aren’t that many
people in this category. The larger danger is those who may be infected but have not yet developed symptoms.

President of Lazyville: The people of our country love watching movies and frequent the cinemas regularly, an activity that acts as a good source of revenue for our government. Setting up movie rental services within the country would cater to such people who would then have easier access to movies. However, setting up such services wouldn’t be of help to the economy since___________.
Which of the following statements most logically completes the argument?
  • a)
    movies that show a lot of violence are a bad influence on the young generation.
  • b)
    the available leisure time is not enough to cater to the increased movie watching and people will cut into their productive time to compensate for it.
  • c)
    nowadays, production of movies has become a costly endeavour with incommensurate returns on investment.
  • d)
    it is cheaper to run a movie rental business than to run a cinema theater.
  • e)
    movie rental services might encourage piracy of movies.
Correct answer is option 'B'. Can you explain this answer?

Disha Mehta answered
Explanation:

Leisure Time Constraints:
Setting up movie rental services within the country would cater to the people who love watching movies and frequent cinemas. However, the argument states that setting up such services wouldn't be of help to the economy. One reason for this is that the available leisure time is not enough to cater to the increased movie watching. People may end up cutting into their productive time to compensate for the increased movie-watching, which could have a negative impact on the economy.
Therefore, option B, stating that the available leisure time is not enough to cater to the increased movie-watching and people will cut into their productive time to compensate for it, most logically completes the argument. This option highlights a potential negative consequence of setting up movie rental services in Lazyville.

A, B, C and D were the members of a team. The average runs of the team decreases by 2 if another member E is added. It is known that E scored 45 runs. No player scored less than E or more than 65 runs. If the runs scored by A and B are in the ratio 13:12 and C scored more than A, what will be the the ratio of the runs scored by B to the average runs scored by C & D ? (Assume that the runs scored by all the members is a natural number).
  • a)
    4:5
  • b)
    5:7
  • c)
    7:9
  • d)
    8:9
  • e)
    16:19
Correct answer is option 'A'. Can you explain this answer?

Notes Wala answered
Let the average runs of the team before adding E be x.
Total runs scored by A and B = 13x + 12x = 25x
Total runs scored by C and D = 2x (since average runs decreases by 2 when E is added)
Total runs scored by the team before adding E = 25x + 2x = 27x
After adding E, the average runs of the team decreases by 2. So, the new average runs = x - 2.
Total runs scored by the team after adding E = (x - 2) * 5
Since E scored 45 runs, we can write the equation:
(x - 2) * 5 = 45
x - 2 = 9
x = 11
So, the average runs before adding E = 11.
Total runs scored by the team before adding E = 27x = 27 * 11 = 297
Now, let's find the individual runs scored by each player.
Since no player scored less than E or more than 65 runs, we can conclude that A scored 65 runs and B scored 65 - 13 = 52 runs.
Let's assume that C scored y runs. Since C scored more than A, y > 65. Also, since y is a natural number, the minimum value of y is 66.
Total runs scored by the team = 65 + 52 + y + 2x = 297
117 + y + 22 = 297
y = 158
So, C scored 158 runs and D scored 297 - 65 - 52 - 158 = 22 runs.
The ratio of the runs scored by B to the average runs scored by C & D = 52 : ((158 + 22)/2) = 52 : 90 = 4 : 5
Therefore, the correct answer is A: 4:5.

In a particular dilution technique, 10% of the solution is removed and replaced with the diluter. If we start with pure alcohol, minimum how many times would the operation need to be performed to bring the percentage of alcohol below 65%.
  • a)
    3
  • b)
    4
  • c)
    5
  • d)
    6
  • e)
    7
Correct answer is option 'C'. Can you explain this answer?

BT Educators answered
Let the total volume of the solution be 100l
The initial concentration of alcohol is 100%.
Thus, the volume of alcohol is 100l.
After first dilution, the volume of the alcohol removed = 10% of 100l = 100*10/100 = 10l
Thus, the volume of alcohol left = 100 - 10 = 90l or 0.9*100
After the second dilution, volume of the alcohol left = [(100-10)/100]*90l = 0.9*90 = 81l
Similarly, after the third dilution, the volume of alcohol left = 0.9*81 = 72.9l.
Thus, after nth dilution, the volume of alcohol left = (0.9)n*100
Since the percentage, if alcohol should be below 65% or 65*100/100 = 65l.
(0.9)n*100 < 65l
(0.9)n < 0.65
When n = 5, (0.9)5 = 0.5905
Thus, after 5 dilutions, the concentration of alcohol will be below 65%.
Thus, the correct answer is C.

A jar contains a mixture of 175 ml water and 700 ml alcohol. Randy takes out 10% of the mixture and substitutes it by water of the same amount. If the process is repeated once again, what will be the percentage of water in the mixture ?
  • a)
    20.5
  • b)
    25.4
  • c)
    29.5
  • d)
    30.3
  • e)
    35.2
Correct answer is option 'E'. Can you explain this answer?

Rahul Kapoor answered
Given that a jar contains a mixture of 175 ml water and 700 ml alcohol.
It is given that 10% of the mixture is removed and it is substituted by water of the same amount and the process is repeated once again
Now we have to find the percentage of water in the mixture.
Since the mixture is removed and substituted with water, we can deal with alcohol and the second step we can find how much amount of alcohol is retained and not about how much amount of alcohol is removed
As 10% of alcohol is removed, 90% of alcohol is retained
So alcohol remaining = 700 × 90% × 90%
⟹ 700 × 0.9 × 0.9 = 567
We totally have 875 ml overall mixture and of this 567 ml is alcohol.
Remaining 875 – 567 = 308 is the amount of water.
We have to find the percentage of water in the mixture i.e. 308/875
Approximately 308 is 30% of 1000 so by this we know that 308 is more than 30%
Hence 35.2% is the percentage of water in the given mixture.
The question is "A jar contains a mixture of 175 ml water and 700 ml alcohol. Gopal takes out 10% of the mixture and substitutes it by water of the same amount. The process is repeated once again. The percentage of water in the mixture is now"
Hence, the answer is 35.2%
Choice D is the correct answer.

A bottle is 80% full. The liquid in the bottle consists of 60% guava juice and 40% pineapple juice. The remainder of the bottle is then filled with 70 mL of rum. How much guava juice is in the bottle?
  • a)
    168 mL
  • b)
    170 mL
  • c)
    200 mL
  • d)
    210 mL
  • e)
    250 mL
Correct answer is option 'A'. Can you explain this answer?

Notes Wala answered
The bottle is filled 80% to capacity, and then 70ml of rum makes the bottle 100% filled to capacity.
So, 70ml represents 20% of the capacity.
So, BEFORE the rum was added, there were 280ml of liquid in the bottle (i.e., if 70ml represents 20% of capacity, then 280ml represents 80% of capacity)
We're told that 60% of the liquid is guava juice.
60% of 280ml = 168 ml

A team of Swedish scientists recently concluded a fifteen year study on the relationship between fatty or lean fish consumption and the risk of kidney cancer; the study revealed that those who ate on average more than one serving per week of fatty fish had 44 percent less risk for developing renal cell carcinoma, the most common form of kidney cancer. Though all previous studies on the relationship between fatty fish and the prevalence of kidney cancer have been inconclusive, the Swedish scientists attribute the lower rate of kidney cancer to increased intake of omega 3 fatty acids. Lean fish is rarely rich in omega 3’s, and those in the study who ate lean fish had the same risk for developing renal cell carcinoma as those who ate no fish at all.
In the statement above, the two portions in boldface play which of the following roles?
  • a)
    The first is an opinion that is supported by the argument; the second is one part of the information uncovered in the study.
  • b)
    The first is a fact that goes against the argument that is being presented; the second is one element of a logical argument in support of the scientists’ claim.
  • c)
    The first presents the quandary the scientists are attempting to solve; the second is the result of that quandary.
  • d)
    The first is a claim in support of the argument; the second is a piece of evidence against the argument.
  • e)
    The first is an explanation advocated by the argument; the second is a finding used to challenge that explanation.
Correct answer is option 'B'. Can you explain this answer?

Wizius Careers answered
In the statement above, the two portions in boldface play which of the following roles?
(A) The first is an opinion that is supported by the argument; the second is one part of the information uncovered in the study.
There is no evidence the first portion is an opinion. And the second part seems to be a fact that was previously known. Wrong.
(B) The first is a fact that goes against the argument that is being presented; the second is one element of a logical argument in support of the scientists’ claim.
The argument is that increasing intake of fatty fish -> lowering rate of kidney cancer. The first BF states that this relationship is inconclusive and hence it goes against the argument or at the least does not support it. The second part uses logic to prove the opposite of consuming fatty fish. Correct
(C) The first presents the quandary the scientists are attempting to solve; the second is the result of that quandary.
The scientists are trying to solve whether fatty fish intake affects rate of kidney cancer.The first BF does not directly present this as a quandary. Wrong
(D) The first is a claim in support of the argument; the second is a piece of evidence against the argument.
As stated above the first BF at best does not support the argument. The second piece at best supports the claim by logically deducing what would happen by consuming non fatty fish. Wrong
(E) The first is an explanation advocated by the argument; the second is a finding used to challenge that explanation.
The first BF does not explain anything but presents information that goes against the conclusion. Wrong

When students receive positive feedback that focuses on getting a question correct or their perceived intelligence, they are less likely to attempt harder assignments than are students who are praised for their effort or the process they used. Because of this phenomenon, teachers who focus on effort and process are more likely to see better learning outcomes than those who focus on whether an individual question was correct or not.

Which of the following is an assumption on which the argument depends?
  • a)
    Students are more likely to learn from doing harder problems than they are to learn from easier ones.
  • b)
    Positive feedback is the best way to motivate students to learn.
  • c)
    Students can clearly articulate the difference between different types of positive feedback.
  • d)
    Feedback based on perceived intelligence is less favorable toward students than are other types of feedback.
  • e)
    The most important indicator of success is whether students are willing to attempt harder questions and assignments.
Correct answer is option 'A'. Can you explain this answer?

Maya Choudhury answered
Assumption Explanation:

Students are more likely to learn from doing harder problems than they are to learn from easier ones.

Explanation:

- The argument suggests that when students receive positive feedback related to their effort or process, they are more likely to attempt harder assignments.
- This implies that students are motivated to take on more challenging tasks when they are praised for their effort or the process they used, rather than just getting a question correct or being seen as intelligent.
- Therefore, the assumption that students are more likely to learn from doing harder problems than easier ones is crucial for the argument's conclusion that teachers focusing on effort and process see better learning outcomes.
- If students were not inclined to learn more from harder problems, the emphasis on effort and process may not necessarily lead to improved learning outcomes.

In conclusion, the assumption that students benefit more from challenging tasks supports the argument's claim that feedback on effort and process leads to better learning outcomes compared to feedback solely based on correctness or perceived intelligence.

A cask is full of wine but it has a leak in the bottom. When one-fourth of the cask empties out because of the leak, the cask is replenished with water. Next when half of the cask has leaked out, it is again filled with water. Finally when three-fourths of the cask leaks out, it is again filled with water. What is the percentage of wine in the cask now?
  • a)
    9.375%
  • b)
    8.33%
  • c)
    7.2%
  • d)
    7.5%
  • e)
    6.66%
Correct answer is option 'A'. Can you explain this answer?

Notes Wala answered
When one-fourth of the cask empties out due to the leak, the cask is replenished with water. This means that 25 units of wine have leaked out, and 25 units of water have been added, leaving 75 units of wine in the cask.
Next, when half of the cask has leaked out, it is again filled with water. At this point, an additional 37.5 units of wine (half of the remaining 75 units) have leaked out, and 37.5 units of water have been added. This leaves us with 37.5 units of wine in the cask.
Finally, when three-fourths of the cask leaks out, it is again filled with water. Another 28.125 units of wine (three-fourths of the remaining 37.5 units) have leaked out, and 28.125 units of water have been added. This leaves us with 9.375 units of wine in the cask.
Now, we need to calculate the percentage of wine in the cask. We divide the remaining units of wine (9.375) by the total capacity of the cask (100) and multiply by 100 to get the percentage:
(9.375 / 100) * 100 = 9.375%
Therefore, the correct answer is A. 9.375%.

There are 100 apples in a bag of which 98% are green and the rest red. How many green apples do you needto remove so that only 96% of the apples are green?
  • a)
    40
  • b)
    8
  • c)
    15
  • d)
    25
  • e)
    50
Correct answer is option 'E'. Can you explain this answer?

Disha Mehta answered
Understanding the Initial Composition
- You have a total of 100 apples.
- 98% of these apples are green, which means:
- Green apples = 98% of 100 = 98 apples
- Red apples = 100 - 98 = 2 apples
Target Composition for Green Apples
- You want to adjust the number of green apples so that they constitute 96% of the total apples remaining after some are removed.
- Let’s denote the number of green apples to be removed as \( x \).
Calculating the New Total of Apples
- After removing \( x \) green apples, the number of green apples becomes \( 98 - x \).
- The total number of apples after removing \( x \) apples is \( 100 - x \).
Setting Up the Equation
- You want the green apples to make up 96% of the new total:
\[
\frac{98 - x}{100 - x} = 0.96
\]
- Cross-multiplying gives:
\[
98 - x = 0.96(100 - x)
\]
- Simplifying the equation:
\[
98 - x = 96 - 0.96x
\]
\[
98 - 96 = x - 0.96x
\]
\[
2 = 0.04x
\]
- Solving for \( x \):
\[
x = \frac{2}{0.04} = 50
\]
Conclusion
- You need to remove 50 green apples for the remaining green apples to constitute 96% of the total apples left in the bag. Thus, the correct answer is option E.

Two solutions of acid were mixed to obtain 10 liters of new solution. Before they were mixed, the first solution contained 0.8 liters of acid while the second contained 0.6 liters of acid. If the percentage of acid in the first solution was twice that in the second, what was the volume of the first solution?
  • a)
    3 liters
  • b)
    3.2 liters
  • c)
    3.6 liters
  • d)
    4 liters
  • e)
    4.2 liters
Correct answer is option 'D'. Can you explain this answer?

Aditya Sharma answered
To solve this problem, we can set up a system of equations based on the given information.

Let's assume the percentage of acid in the second solution is x%. According to the problem, the percentage of acid in the first solution is twice that of the second, so it would be 2x%.

Let's calculate the amount of acid in each solution:
Amount of acid in the first solution = 0.8 liters * (2x/100) = 1.6x/100 liters
Amount of acid in the second solution = 0.6 liters * (x/100) = 0.6x/100 liters

The total amount of acid in the new solution is the sum of the acid in the first and second solutions, which is equal to 10 liters:
1.6x/100 + 0.6x/100 = 10

To simplify the equation, we can multiply both sides by 100 to get rid of the denominators:
1.6x + 0.6x = 1000
2.2x = 1000
x = 1000/2.2
x ≈ 454.55

So, the percentage of acid in the second solution is approximately 454.55%.

Now, let's find the volume of the first solution.
We know that the volume of the first solution is 0.8 liters and the percentage of acid in it is 2x% ≈ 909.09%.

Therefore, the answer is 0.8 liters (option D).

In summary:
- The percentage of acid in the second solution is approximately 454.55%.
- The volume of the first solution is 0.8 liters.

If 1 cup of water is added to a 5-cup mixture that is 2/3 salt and 1/3 water, what percent of the 6-cup mixture is salt?​
  • a)
    20%​
  • b)
    25%​
  • c)
    33.33%​
  • d)
    55.56%​
  • e)
    66.67%​
Correct answer is option 'D'. Can you explain this answer?

Prateek Gupta answered
Understanding the Mixture
To solve this problem, we first need to analyze the initial 5-cup mixture. The composition is given as 2/3 salt and 1/3 water.
Calculate the Amount of Salt and Water in the Mixture
- Total volume of the mixture = 5 cups
- Volume of salt = (2/3) * 5 = 3.33 cups
- Volume of water = (1/3) * 5 = 1.67 cups
Add Water to the Mixture
Next, we add 1 cup of water to the existing mixture.
- New volume of water = 1.67 cups + 1 cup = 2.67 cups
- Total volume of the new mixture = 5 cups + 1 cup = 6 cups
Calculate the Total Salt in the New Mixture
- The amount of salt remains unchanged at 3.33 cups.
Calculate the Percentage of Salt in the New Mixture
To find the percentage of salt in the new 6-cup mixture, use the formula:
\[ \text{Percentage of Salt} = \left( \frac{\text{Volume of Salt}}{\text{Total Volume}} \right) \times 100 \]
- Volume of salt = 3.33 cups
- Total volume = 6 cups
Plugging in the values:
\[ \text{Percentage of Salt} = \left( \frac{3.33}{6} \right) \times 100 \approx 55.56\% \]
Conclusion
Thus, the percentage of salt in the 6-cup mixture is approximately 55.56%, confirming that the correct answer is option 'D'.

An incredible punch is composed of buttermilk, orange juice, and brandy. How many pints of orange juice are required to make 7 1⁄2 gallons of punch containing twice as much buttermilk as orange juice and three times as much orange juice as brandy? (1 Gallon = 8 Pints )
  • a)
    16
  • b)
    18
  • c)
    20
  • d)
    22
  • e)
    24
Correct answer is option 'B'. Can you explain this answer?

Rahul Kapoor answered
Let's assume the amount of orange juice needed is x pints.
According to the given information:
  • The punch contains twice as much buttermilk as orange juice, so the amount of buttermilk required is 2x pints.
  • The punch contains three times as much orange juice as brandy, so the amount of brandy required is x/3 pints.
Now, we can set up an equation based on the total volume of the punch:
2x + x + x/3 = 7.5 * 8
2x + x + x/3 = 60
To simplify the equation, we'll multiply everything by 3 to eliminate the fraction:
6x + 3x + x = 180
10x = 180
x = 18
Therefore, 18 pints of orange juice are required to make 7 1/2 gallons of punch.
Since 1 gallon is equal to 8 pints, 7 1/2 gallons would be equal to 7.5 * 8 = 60 pints.
Now, we need to determine the amount of orange juice required, which is 18 pints.
The answer choice that corresponds to 18 pints is (B) 18.

Commentator: In the new century, only nations with all the latest electronic technology will experience great economic prosperity. The people in these nations will be constantly bombarded with images of how people in other countries live. This will increase their tendency to question their own customs and traditions, leading to a dissolution of those customs and traditions. Hence, in the new century, the stability of a nation’s cultural identity will likely ______________ .
Which one of the following most logically completes the commentator’s argument?
  • a)
    depend on a just distribution of electronic technology among all nations
  • b)
    decrease if that nation comes to have a high level of economic wealth
  • c)
    be ensured by laws that protect the customs and traditions of that culture
  • d)
    be threatened only if the people of that culture fail to acquire the latest technical skills
  • e)
    be best maintained by ensuring the gradual assimilation of new technical knowledge and skills
Correct answer is option 'B'. Can you explain this answer?

Wizius Careers answered
The commentator's argument discusses the impact of advanced electronic technology on the stability of a nation's cultural identity in the new century. To identify the option that most logically completes the argument, let's analyze each answer choice:
A) Depend on a just distribution of electronic technology among all nations:
  • This option introduces the concept of a fair distribution of electronic technology among nations.
  • However, the argument does not focus on the distribution of technology, but rather on the impact of technology on cultural identity.
  • Thus, this choice is not directly related to the commentator's argument.
B) Decrease if that nation comes to have a high level of economic wealth:
  • This option suggests that the stability of a nation's cultural identity will decrease if the nation achieves a high level of economic wealth.
  • The argument discusses how exposure to advanced electronic technology can lead people to question their own customs and traditions.
  • Economic wealth can enable the adoption of advanced technology and exposure to other cultures, increasing the likelihood of questioning customs and traditions.
  • Therefore, this option aligns with the commentator's argument and logically completes it.
C) Be ensured by laws that protect the customs and traditions of that culture:
  • This option proposes that the stability of a nation's cultural identity can be ensured through laws protecting customs and traditions.
  • While laws can play a role in preserving cultural identity, the argument primarily focuses on the influence of technology and exposure to other cultures.
  • Therefore, this choice does not directly address the commentator's argument.
D) Be threatened only if the people of that culture fail to acquire the latest technical skills:
  • This option suggests that the stability of a nation's cultural identity is threatened only if the people of that culture fail to acquire the latest technical skills.
  • The argument focuses on the impact of exposure to advanced technology and images of other cultures, not solely on technical skills.
  • Thus, this option is not the most logical completion of the argument.
E) Be best maintained by ensuring the gradual assimilation of new technical knowledge and skills:
  • This option proposes that the cultural identity of a nation is best maintained by gradually assimilating new technical knowledge and skills.
  • The argument discusses how exposure to technology and images of other cultures can lead to questioning of customs and traditions.
  • Gradually assimilating new technical knowledge and skills may contribute to the erosion of cultural identity, as the argument implies.
  • Therefore, this choice does not align with the commentator's argument.
Based on the analysis, the option that most logically completes the commentator's argument is (B) decrease if that nation comes to have a high level of economic wealth. This choice supports the argument's assertion that exposure to advanced electronic technology, facilitated by economic wealth, can lead to the dissolution of customs and traditions.

Due to significant advances in biotechnology, experts predict that within years, doctors will be able to trace the genetic roots of common medical problems such as depression and bi-polar syndrome. As a result, some physicians predict that these conditions and others like them will be all but eliminated through early identification and genetic therapy.
Q. The argument above is based most heavily upon which of the following assumptions?
  • a)
    There is one and only one strategy for eliminating common medical problems such as depression.
  • b)
    Common medical problems such as depression are based entirely on treatable genetic malformations.
  • c)
    Within years, genetics will be the only important scientific field in detecting and treating common medical problems such as depression.
  • d)
    Every member of the human race has at least one genetic defect.
  • e)
    Each human defect or sickness can be traced in part to genetics.
Correct answer is option 'B'. Can you explain this answer?

Amrutha Jain answered
The argument concludes that "common medical problems such as depression" "will be all but eliminated through early identification and genetic therapy." This is based upon the assumption that the only cause of these common medical problems is a treatable genetic deformity. If this assumption proves false (and problems arise from non-genetic issues), the argument's conclusion is not logical as the genetic therapy would not stop common medical problems that were caused by non-genetic factors.
A. The original argument does not pertain to the number of treatments for common medical problems but rather pertains to the ability of early identification and genetic therapy to eliminate common medical problems. Consequently, whether there is only one method (the one described) or one million methods is irrelevant.
B. This answer points out that if common medical problems arise either in part or in whole for reasons that are not genetic, early genetic detection and therapy will not rectify the problem. The assumption that these common medical problems are based entirely on treatable genetic malformations is an essential part of the argument.
C. This may be a logical result (or consequence) if the original argument proves to be true. However, it is not a necessary assumption for the original argument to be true.
D. The original argument pertains to the ability of doctors to treat "common medical problems" when they exist not whether these "common medical problems" exist in everyone. Further, the original argument hinges on the assumption that these problems are rooted in genetic defects and treatable via genetic therapy.
D. The original argument pertains to common medical problems such as depression and not to "each human defect or sickness." Further, the original argument assumes that genetic therapy alone is sufficient to treat the problem while this answer simply states that all human defects or sickness can be traced in part to (and not necessarily treated by) genetics.

A recent article in one of the nation's leading newspapers noted that despite the government's warning about peanut butter likely being contaminated by salmonella and the government's subsequent recall of a limited amount of peanut butter, 90% of grocery store shoppers surveyed said that they did not plan to change their peanut butter purchasing habits. Nevertheless, roughly two months after the limited recall and one month after the leading newspaper published its article, the country's peanut butter manufacturers reported that same-store sales to grocery store shoppers fell 75% year-over-year.
Q. Which of the following, if true, best explains the apparent paradox above?
  • a)
    The initial survey of shoppers failed to consider the effect of subsequent cuts in the price of peanut butter.
  • b)
    Fearing additional instances of contamination and subsequent lawsuits, many retailers that sold peanut butter removed the product voluntarily from their shelves.
  • c)
    A report similar to the report that appeared in the leading newspaper appeared in one of the nation's tabloid magazines on the same day.
  • d)
    Days before the newspaper conducted its survey, a widely-respected bacterial research specialist published an op-ed article in a major newspaper arguing that the threat from salmonella-infected peanut butter was smaller than the government would later contend.
  • e)
    A study published after the government recall of some peanut butter stated that individuals intended to change the type of jelly and bread they purchased.
Correct answer is option 'B'. Can you explain this answer?

Anirban Das answered
The paradox: After a government recall of some peanut butter, shoppers indicated that they would not change their peanut butter purchasing habits. However, same-store-sales of peanut butter subsequently fell dramatically.
A.This answer heightens the paradox by making the subsequent fall in sales of peanut butter even more unexplainable since a price cut would have stimulated sales (not provided an explanation for their decline).
B. This answer identifies a correct explanation for the paradox. With stores fearful about lawsuits, they removed peanut butter and consequently "same-store sales to customers fell."
C. Although a tabloid is a much less respected source than a leading newspaper, the fact still remains that the article appeared in a leading newspaper. Further, the source of the news does not explain the discrepancy between individuals' stated intent to continue purchasing peanut butter and the subsequent poor sales figures. If the tabloid as a source were a problem, it would only further increase the paradox over why sales declined (why would consumers seemingly base their decisions upon a tabloid).
D. When the individuals took the survey where they stated their intent to continue purchasing peanut butter, they knew about the op-ed piece as it appeared "days before the newspaper conducted its survey." Consequently, the op-ed cannot explain individuals' switch in behavior (i.e., intending to purchase peanut butter but then deciding not to).
E. The original argument notes that "the country's peanut butter manufacturers" (i.e., every manufacturer accounted for—not limited to the specific type of peanut butter recalled) reported a drop in sales. Although consumers' intention to change the type of jelly and bread they purchased could imply that consumers would change the type of peanut butter they purchased, it does not explain why sales would virtually stop altogether. There is a difference between changing the type of jelly purchased (and by correlation, the type of peanut butter purchased) and stopping purchasing jelly altogether (and by correlation, stopping purchasing peanut butter altogether).

Simba borrowed $12,000 from his brothers so he can buy a new sports car. If Simba returns 4.5% of that amount every 2 weeks, after how many months Simba wouldn’t owe his brothers any more money?
  • a)
    8
  • b)
    12
  • c)
    15
  • d)
    18
  • e)
    20
Correct answer is option 'B'. Can you explain this answer?

Nikhil Khanna answered
To find out how many months Simba will take to repay the loan, we need to calculate how many 2-week periods are in a month. Since there are 52 weeks in a year, and 12 months in a year, there are 52/12 = <52 2="4.33">>4.33 2-week periods in a month.
Simba returns 4.5% of the loan amount, which is $12,000 x 4.5/100 = $<12000*4.5 00="540">>540 every 2 weeks.
Therefore, Simba repays $540 x 4.33 = $<540*4.33=2338.2>>2338.2 every month.
To find out how many months Simba will take to repay the loan, we divide the loan amount by the monthly payment: $12,000 / $2338.2 = 5.13 months.
Therefore, Simba will take approximately 5.13 months to repay the loan. Answer: \boxed{5}.

Eating beets significantly lowers the risk of cancer, according to an article in a nutritional magazine. The article refers to a study that found that people who consumed one or more beets per day were half as likely to be diagnosed with the disease as people who did not.
Q. Which of the following, if true, most weakens the argument in the magazine article?
  • a)
    Another study found that people who consumed one tablespoon of flax seed oil per day were more than four times less likely to be diagnosed with cancer as those who did not.
  • b)
    Participants in the study reported consuming no vegetables other than beets.
  • c)
    The study was only conducted in one city.
  • d)
    In another experiment, cancer patients who ate one or more beets per day were no more likely to recover than those who ate no beets.
  • e)
    The participants in the study who ate beets were more likely to exercise regularly than those who did not eat beets.
Correct answer is option 'E'. Can you explain this answer?

Devansh Shah answered
The article draws a conclusion on the basis of the findings of a single study. However, the link between eating beets and cancer prevention cannot be verified without ruling out other variables. If the participants who ate beets were also similar in another way that distinguished them from the group that did not, it would become impossible to prove that beets - and not the other factor - were responsible for reducing the risk of cancer on the basis of this study alone.
A. The effectiveness of flax seed oil in reducing the risk of cancer has no bearing on the effectiveness of beets in doing so. This answer is off topic.
B. This answer strengthens, rather than weakens, the argument. If the subjects ate only beets and no other vegetables, there is more evidence for the fact that beets – and no other vegetable – were responsible for reducing the risk of cancer.
C. The study would be more convincing if it had been conducted in more than one city. However, this fact alone does not do the most to weaken the argument.
D. This other experiment is about the role of beets in the recovery rates of cancer patients, rather than in the risk of diagnosis. Therefore, it does not weaken the conclusion of the article, which focuses on cancer prevention, not recovery.
E. Because study participants who ate beets were also more likely to exercise regularly than those who did not eat beets, it is impossible to determine whether beets or regular exercise were more influential in preventing cancer. This is the correct answer.

Virtually all health experts agree that second-hand smoke poses a serious health risk. After the publication of yet another research paper explicating the link between exposure to second-hand smoke and a shorter life span, some members of the State House of Representatives proposed a ban on smoking in most public places in an attempt to promote quality of life and length of lifespan.
Q. Which of the following, if true, provides the most support for the actions of the State Representatives?
  • a)
    The amount of damaging chemicals and fumes released into the air by cigarette smoke is far less than the amount released from automobiles, especially from older models.
  • b)
    Banning smoking in most public places will not considerably reduce the percent of the population in the state in question that smokes.
  • c)
    The state whose legislators are proposing the tough smoking legislation has a relatively high percent of its population that smoke.
  • d)
    Another state that enacted a similar law a decade ago saw a statistically significant drop in lung-cancer rates among non-smokers.
  • e)
    A nearby state up-wind has the highest number of smokers in the country.
Correct answer is option 'D'. Can you explain this answer?

The State Representatives' argument for banning smoking is based upon scientific research and the presence of a correlation between second-hand smoke and life expectancy. However, it could be strengthened if data existed to show that other regions that enacted tough anti-smoking reform experienced longer life spans. In other words, although we know there is a relationship between second-hand smoke and life expectancy, we do not know that enacting tough anti-smoking reform will influence second-hand smoke levels and thereby influence life expectancy.
A. The legislators' argument is about protecting people from second-hand smoke, not about taking one action versus another (i.e., the legislators are not comparing sources of toxin, but rather attempting to prevent one source from entering the air).
B. The question at hand does not pertain to the percent of the population that smokes but the ability of the law to extend life expectancy. This answer fails to make a connection between the proposed law and extending life expectancy.
C. Although the percent of the state population that smokes will affect the extent of the impact made by the law, it does not support the merits of the law in and of itself. In other words, the argument is not based upon the number or percent of the population that smokes (and by corollary the number and percent of the population affected by second-hand smoke). Rather, the argument is based upon a connection between removing second-hand smoke inhalation via legislation and lengthening life span. This answer provides no direct evidence to strengthen the link between removing second-hand smoke via legislation and lengthening lifespan.
D. The evidence that passing a similar law reduced cancer rates supports the legislators' case that banning smoking in many places will promote "length of lifespan" (i.e., with people dying of cancer less, they live longer).
E. The number of smokers in a nearby state does not influence whether banning second-hand smoke in the state in question will affect life expectancy. The large number of smokers up-stream will hurt air quality and length of life downstream (weakling the legislators' argument if it effected it at all). Fundamentally, this answer is wrong because it fails to strengthen the connection between removing second-hand smoke via legislation and lengthening life-expectancy.

Linda has the choice to buy raw materials from one of two vendors. Vendor X offers a 5% bulk order discount but includes a handling fee with each purchase. Vendor Y doesn't offer a discount but doesn't charge any additional fees. The handling fee Vendor X charges is $10 for orders under $250, $20 for orders from $250 to $600, and $25 for orders above $600. Linda calculates that for very small orders, Vendor Y is more cost-effective, but she suspects that for larger orders, Vendor X might be cheaper.
I. Determine the smallest value of an order from Vendor Y for which an identical order from Vendor X would be less expensive. 
II. If an order costs $500 from Vendor Y, how much would the same order cost from Vendor X?
  • a)
    $500, $475
  • b)
    $400, $485
  • c)
    $210, $475
  • d)
    $400, $380
  • e)
    $210, $200
Correct answer is option 'D'. Can you explain this answer?

Pranav Das answered
Analysis:
Vendor X offers a 5% bulk order discount but charges a handling fee, while Vendor Y does not offer a discount but does not charge any additional fees.

I. Smallest Value of an Order:
To determine the smallest value of an order from Vendor Y for which an identical order from Vendor X would be less expensive, we need to calculate the crossover point where the total cost from each vendor is the same.

II. Calculation for a $500 Order:
For an order costing $500 from Vendor Y:
- Vendor X's cost would be $500 - 5% discount + handling fee
- Cost of order from Vendor X = $500 - 5% of $500 + handling fee
To find the exact crossover point where Vendor X becomes cheaper than Vendor Y, we need to compare the total cost for both vendors at different order values.

Calculation:
- For orders under $250, Vendor Y is cheaper
- For orders from $250 to $600, we need to find the exact crossover point
- For orders above $600, Vendor X is cheaper

Answer Explanation:
- The correct answer is option D: $400 from Vendor Y would cost $380 from Vendor X.
- For an order of $500, Vendor X would not be cheaper than Vendor Y.
- By comparing the costs at different order values, we find that the crossover point is at $400, where Vendor X becomes cheaper than Vendor Y.

Samuel is obviously a bad fisherman. During the past season, in which he and the five members of his team spent four months on a boat together off Dutch Harbor, AK, he caught fewer fish than any of his teammates.
Q. Which of the following, if true, most weakens the argument above?
  • a)
    Two seasons ago, Samuel fished on another boat off Dutch Harbor and caught more fish than any other member of that boat.
  • b)
    Before becoming a fisherman, Samuel piloted a fishing boat whose members regularly caught record numbers of fish.
  • c)
    While fishing this past season, Samuel fell sick for a week and did not catch any fish during this time.
  • d)
    Unlike the other fishermen on his boat, at the order of the captain, Samuel fished this past season with experimental bait.
  • e)
    Amongst the fishing community in Dutch Harbor, Samuel has a reputation for being an especially bad fisherman.
Correct answer is option 'D'. Can you explain this answer?

Aditya Sharma answered
The conclusion to the argument is that "Samuel is obviously a bad fisherman" while the premise is Samuel's poor fishing performance relative to the peers on his fishing boat this past season.
A. The same logical flaws are at play in this answer (except in reverse). Using one season and a comparison to the fishermen on one boat does not provide a wide enough basis to make a judgment about a fishermen's ability relative to all other fishermen. Comparing this past season with another season still does not help to explain Samuel's poor performance this season (whereas knowing that Samuel used experimental bait this season would provide a justification for why Samuel caught few fish yet was still not a terrible fisherman).
B. Samuel's performance as a pilot does not relate to his ability as a fisherman. This answer is off topic.
C On first glance, this looks like a good answer. However, it does not consider whether Samuel's teammates were sick during the same period. Perhaps one of Samuel's teammates was sick for two weeks.
D. This answer highlights something that made Samuel's fishing performance uniquely different than his teammates. Moreover, Samuel did not make the choice to fish with alternative bait--his captain ordered him to do it. If Samuel made the choice to fish with alternative bait, it would be his poor fishing judgment that would be at fault. Similarly, the answer makes clear that no other fisherman on Samuel's boat faced the same predicament.
E. If anything, this answer strengthens the argument.

In an attempt to abate the pernicious decline in MicroChip's revenue brought about by shrinking demand that is accompanying an economic recession, MicroChip is offering customers a 50% discount for the next three months on all purchases fully paid for within 15 days.
Q. Which of the following assumptions most underlies the chip maker's offer of a discount?
  • a)
    MicroChip expects this discount to help the company retain existing customers and gain new ones, enabling the firm to survive in the long-term.
  • b)
    There are no other competing chip companies with prices lower than the reduced price.
  • c)
    The government will provide massive technology tax credits to businesses, spurring them to purchase chips and other related products.
  • d)
    The government will not pursue MircoChip if in fact its behavior in offering a deep discount amounts to a violation of predatory pricing laws.
  • e)
    The decrease in revenue brought about by the reduction in price will be smaller than the anticipated increase in revenue brought about by the increase in demand (spurred by the reduction in price).
Correct answer is option 'E'. Can you explain this answer?

Palak Yadav answered
In answering this question, we must keep separate revenue and profit. Further, we must separate revenue from all other aspects of the business. It is important to keep in mind that MicroChip's efforts are aimed at increasing revenue so any assumption that does not pertain to revenue is almost certainly unnecessary.
A. This answer addresses "enabling the firm to survive in the long-term." However, the original argument deals with "an attempt to abate the pernicious decline in MicroChip's revenue." In other words, this answer deals with the ability of the firm to survive in the long-term and retain customers instead of addressing how the company will increase its revenue (which is different from the number of customers or the level of profitability). It is not necessary to make an assumption about long-term survival in order to make a conclusion about increasing revenue.
B. Even if there were other companies with lower prices, as long as the other companies do not further lower their prices, it will not affect MicroChip's ability to increase its demand and revenue. Further, the original argument only deals with MicroChip's attempt to increase its revenue. In other words, the original argument deals with MicroChip's ability to raise its revenue on absolute terms→not its ability to raise revenue more than rivals. It is not necessary to make an assumption about other firms' prices in order to make a conclusion about increasing MicroChip's revenue.
C. Although this would improve MicroChip's revenue, it is not a necessary assumption underlying MicroChip's strategy of offering a discount as the company did not need to offer a discount to increase revenue if it assumed the government would stimulate demand on its own. In other words, if MicroChip assumed the government would stimulate demand, MicroChip would not have offered a discount for the discount would not have been necessary.
D. This answer does not address an assumption that underlies increasing revenue, but rather legality. Further, the answer does not state that MicroChip's actions violate a law. Finally, even if the government pursued charges, this would not necessarily decrease revenue (although it would certainly decrease profit as MicroChip would incur legal fees etc.).
E. This answer contains the assumption that directly relates to MicroChip's attempt to increase its revenue. In order for the decrease in price to increase total revenue, the volume of sales must increase as total revenue equals price multiplied by volume.

Article 1
From a local Southern California newspaper.

Typically, fewer people vote in local elections that in national ones, but participation can vary widely by state. A new poll has found that more people in California vote in local elections than do people in Nevada, Arizona, and New Mexico combined. This has been attributed to the high number of independent voters in our state, as well as the high enthusiasm for local elections. City councilman Brad Zellman was pleased to see such a high turnout at the recent city council election. “It’s a unique opportunity for local citizens’ votes to carry more weight than they would in big national elections, and it’s great to have so much participation in these smaller campaigns state-wide.”
 
Article 2
Editorial from the Opinion section of a competing newspaper.

The recent city council election has seen many a politico claim that the increased voter turnout is due to local citizens wanting to be a part of an election where their vote is proportionally more important. However, it is incorrect to suggest that voting in local elections somehow means your vote “counts more.” True, local politicians make decisions that affect day-to-day operations of our cities, but national politicians make decisions that ultimately affect the outcomes of our lives. When you vote for national figures in larger elections, your vote matters more because you are voting for people who will ultimately have a bigger impact on your life long-term.
 
Article 3
Results from recent California elections.

City councilman elections: 13,000 local votes cast (22% of the eligible population voted), 80% of citizens who voted were polled as “extremely satisfied” with the results. District representative elections: 1.9 million votes cast (19% of the eligible population voted), 67% of citizens who voted were polled as “extremely satisfied” with the results.
Consider each of the following statements. Does the information in the articles support the inference as stated?
Californians are more concerned with politics than are people in Nevada, Arizona, and New Mexico.
The “weight” Zellman describes refers to the scale of political decision-making.
 
  • a)
    Yes, Yes
  • b)
    No, No
  • c)
    Yes, No
  • d)
    No, Yes
  • e)
    Cannot be determined
Correct answer is option 'B'. Can you explain this answer?

EduRev GMAT answered
Question 1 Explanation: The answer is No. This argument assumes that the percentage of people who vote in California is higher than the percentage of people who vote in the other three states put together, based on the fact that the number of people who vote is greater in California. We cannot infer anything about the percentage of people who vote based only on actual numbers of people who vote.
Question 2 Explanation: The answer is No. Zellman likely means that your vote carries “more weight” in local elections since “typically fewer people vote in local elections” according to Article 1. This is further established by Article 2, which begins with a refutation of Zellman and the idea that a vote in a local election “counts more.”
 

After studying a random sample of 1024 individuals who had smoked daily for at least three years and comparing the results of this study with the results of a study of 1024 individuals who had never smoked, a group of researchers concluded that habitual smoking causes increased difficulties in concentrating.
Q. Which of the following, if true, most severely weakens the researchers' conclusion?
  • a)
    The addiction to smoking and the cravings this addiction engenders is often on the mind of habitual smokers.
  • b)
    Some non-smokers with attention deficit disorder (ADD), which causes an inability to concentrate, display even less ability to concentrate than some smokers.
  • c)
    A separate research study found that smokers and non-smokers exhibited statistically significant differences in their incarceration rates.
  • d)
    After developing a severe addiction to smoking for fifteen years, the ability of many individuals to concentrate is decreased.
  • e)
    A separate research study found that individuals with preexisting attention and concentration disorders exhibited significantly higher rates of trying cigarettes and subsequently becoming addicted to smoking.
Correct answer is option 'E'. Can you explain this answer?

Aditya Gupta answered
The researchers' conclusion is: "habitual smoking causes increased difficulties in concentrating"
The group of researchers confuses correlation with causation. In other words, the group concludes that a correlation between smoking and an inability to concentrate implies that smoking causes an inability to concentrate. Perhaps it is the other way around and an inability to concentrate causes individuals to become distracted and take up smoking. Evidence to support this counter theory would weaken the researchers' conclusion.
A. This answer strengthens (not weakens) the original argument as it provides an explanation for how habitual smoking increases difficulties in concentrating after becoming hooked.
B. The answer deals only with some ADD non-smokers and some smokers so it does not provide any solid evidence and justification to weaken or reject the original argument. Further, it is not logical to compare the concentration ability of individuals with an attention deficit condition to other individuals who do not have an attention deficit condition.
C. The incarceration rate is irrelevant in determining the relationship between concentration and smoking. Whether smokers are incarcerated at higher or lower rates does not enable us to strengthen or weaken the causal relationship between smoking and concentration proposed in the original argument.
D. This strengthens (not weakens) the original argument as it intensifies the causal relationship between smoking and subsequent difficulties concentrating.
E. This additional study pinpoints that individuals with preexisting (or already established) concentration difficulties subsequently became addicted to smoking. This pinpoints that the smoking could not have caused the attention and concentration difficulties (as these difficulties already existed prior to becoming addicted).

As a result of implementing an experimental farming system that combined aggressive new fertilization, deep irrigation, and speculative pesticides, the yield on crops at a farm in central California grew consistently and considerably over the past six years. However, yields this year unexpectedly plummeted, causing the farm's owners considerable financial difficulties.
Q. Which of the following statements, if true, best explains the unexpected drop in yield?
  • a)
    As a result of a serious and recent economic recession, the market for the products the farm produced shrank in size during the past year.
  • b)
    Other farms that began using the experimental system at about the same time also reported an unexpected plummet in yields this year.
  • c)
    The central California region where the farm in question is located experienced a drought 9 years ago, 3 years ago, and this past year.
  • d)
    A different experimental system, used on a different crop, failed after two years at a farm in Iowa. Scientists later discovered that chemicals involved in this different system slowly poisoned the soil.
  • e)
    Significant and sustained increases in agricultural productivity eventually lead to an exhaustion of important nutrients in land.
Correct answer is option 'E'. Can you explain this answer?

The yield at a central Californian farm dropped significantly after 6 years of consistent growth. Clearly, something of importance to the yield changed. Some possibilities include: (1) a storm decimated the crop (although this level of storm could have only hit this year, as the crops grew "consistently and considerably over the past six years") (2) the farmer stopped using the technique (3) over a period of six years, the technique overused natural soil nutrients and drained important chemicals that the farm crops needed to grow.
A. The original argument pertains to a decline in the yield of the farm, not a decline in the buyers. The farm's yield has no relationship with the number of buyers as yield simply refers to the ratio of seeds planted to crops available for harvest.
B. Although this confirms that the drop the farm in central California experienced is not unique, it does not explain the drop in yield.
C. Since a drought occurred 9 years ago prior to the experimental system and during the experimental system (3 years ago) without any affect on yields, it is not logical to conclude that the drought caused the decline in yields this year. In other words, since the yield grew "consistently" even during the drought 3 years ago and 6 years ago, it is illogical to conclude that the drought caused the decline in yield this year.
D. The circumstances between the two experimental systems are too different to allow comparisons and a logical deduction that the errors of the Iowa system explain the errors of the central California system. Specifically: (1) the length of time before declines in yields occurred differed considerably (2) the crops the farm produced differed (3) the geographic region of the farm (and climate) differed considerably.
E. The sustained (6 years, "consistently") and significant ("considerably") expansion in productivity ("yield") led to exhaustion of nutrients, meaning crops could not attain the chemical compounds they needed to grow. This answer explains why the crop yield dropped after so many years and why it dropped after years of growth.

Authors writing detective stories frequently include a brilliant detective and an incompetent investigator who embark on separate paths in an attempt to solve a crime. The separate accounts frequently consist of the incompetent investigator becoming distracted by the criminals' well-planned attempts and the competent detective solving the case after a violent confrontation. Many literary analysts believe authors often choose this storyline in an attempt to provide readers additional complexity and challenge in solving the investigation.
Q. Which of the following most logically follows from the statements above?
  • a)
    A well-written detective story consists of an investigation being undertaken by a competent and incompetent investigator.
  • b)
    Some authors use an incompetent investigator to show the complexities of an investigation.
  • c)
    Authors never write stories with incompetent investigators who solve a case correctly.
  • d)
    Authors can use the separate investigative accounts to make predicting the correct outcome of the investigation more difficult.
  • e)
    Authors write stories with competent and incompetent investigators to show the complexity of real life.
Correct answer is option 'D'. Can you explain this answer?

Advait Malik answered
This question asks you to take the statements and draw a conclusion. One major trap in this type of question is an answer that is logical, but not supported by the statements in the stimulus.
A. The stimulus does not define what constitutes a well written story nor does it speak about what is a poorly written story. Consequently, it is difficult to make a statement like this that will logically follow from the stimulus.
B. Although this statement is probably true, it does not follow from the stimulus. Instead, the stimulus states that authors use an incompetent investigator to add complexity to the storyline (thereby making the reader's attempts to solve the case more challenging) not to show that investigations are complex.
C. While the stimulus states that stories "frequently" include an incompetent investigator who does not solve the case correctly, we cannot conclude an incompetent investigator "never" solves a case correctly.
D. This statement is quite similar to the final sentence of the stimulus and it logically follows from the stimulus. The statement that authors write in the way they do "to provide readers additional complexity and challenge in solving the investigation" provides the basis to conclude that authors write "to make predicting the correct outcome of the investigation more difficult."
E. Although this statement is probably true, there is no evidence of it in the stimulus. Instead, the stimulus indicates that the complexity is added not for its resemblance to real life but for its ability to increase the challenge posed to readers seeking to solve the case correctly.

The vessels contain water and milk in the ratio 1:2 and 2:5 are mixed in the ratio 1:4. The resulting mixture will have water and milk in the ratio.
  • a)
    31:74
  • b)
    31:75
  • c)
    30:77
  • d)
    30:74
  • e)
    31:77
Correct answer is option 'A'. Can you explain this answer?

Given:
- The vessels contain water and milk in the ratio 1:2
- The vessels contain water and milk in the ratio 2:5
- The mixture ratio is 1:4

To Find:
The ratio of water and milk in the resulting mixture.

Solution:
Let's assume the initial quantities of water and milk in the first vessel are 1x and 2x respectively.
Similarly, the initial quantities of water and milk in the second vessel are 2y and 5y respectively.

Mixing the Vessels:
When the two vessels are mixed in the ratio 1:4, the resulting mixture will have a total of 1+4=5 parts.

Calculating the Quantities of Water and Milk:
1. In the first vessel, the ratio of water to milk is 1:2. So, the total quantity of the mixture in the first vessel is 1x+2x=3x.
2. In the second vessel, the ratio of water to milk is 2:5. So, the total quantity of the mixture in the second vessel is 2y+5y=7y.

Calculating the Ratio of Water:
1. In the first vessel, the ratio of water to the total mixture is 1:3 (1x:3x).
2. In the second vessel, the ratio of water to the total mixture is 2:7 (2y:7y).

Calculating the Overall Ratio of Water:
For the resulting mixture, the ratio of water to the total mixture is 1:5 (1:4+1).
So, the ratio of water to the total mixture can be calculated by combining the ratios from both vessels:
(1x/3x + 2y/7y) / 2 = 1/5

Cross-multiplying:
7(1x) + 3(2y) = 2(3x)(5)
7x + 6y = 30x
6y = 23x
y/x = 23/6

Substituting the Value of y/x:
(1x/3x + 2(23/6)x/7(23/6)x) / 2 = 1/5
(1/3 + 46/84) / 2 = 1/5
(28/84 + 46/84) / 2 = 1/5
74/84 / 2 = 1/5
74/168 = 1/5

Simplifying the Ratio:
Dividing both numerator and denominator by 4:
(74/4)/(168/4) = 31/77

Conclusion:
Therefore, the resulting mixture will have water and milk in the ratio 31:77.

Paul walks from home to work at a rate of 5 mph and bikes home from work along the same route at 12 mph. What is his average speed for the round trip?
  • a)
    7/2
  • b)
    90/17
  • c)
    120/17
  • d)
    17/2
  • e)
    9
Correct answer is option 'C'. Can you explain this answer?

Moumita Sen answered
Understanding Average Speed
To find the average speed for the entire round trip, we need to consider both the distance and the time taken for each leg of the journey.
Distance Calculation
- Let the one-way distance from home to work be "d" miles.
- Therefore, the total round trip distance is:
Total Distance = d (to work) + d (back home) = 2d miles
Time Calculation
- Time taken to walk to work:
Time (walk) = Distance / Speed = d / 5 hours
- Time taken to bike home:
Time (bike) = Distance / Speed = d / 12 hours
- Total Time for the round trip:
Total Time = Time (walk) + Time (bike) = d/5 + d/12
Finding a Common Denominator
- The least common multiple of 5 and 12 is 60.
- To combine the times:
d/5 = 12d/60
d/12 = 5d/60
- Therefore, Total Time = (12d/60) + (5d/60) = (17d/60) hours
Average Speed Formula
- The average speed (S) is given by the formula:
Average Speed = Total Distance / Total Time
- Substituting the values:
Average Speed = 2d / (17d/60)
Simplifying the Average Speed
- The "d" cancels out:
Average Speed = 2 / (17/60) = 2 * (60/17) = 120/17 mph
Conclusion
Thus, the average speed for Paul’s round trip is 120/17 mph, which corresponds to option 'C'.

If X is a root of the equation a3 +8a2 – 20a, than which of the following equations Don’t have the root X as one of their roots?
  • a)
    X3 + 4X2 – 32X
  • b)
    X2 + 18X + 80
  • c)
    X2 – 12X + 20
  • d)
    X2 + 5X – 14
  • e)
    X2 + 10X + 16
Correct answer is option 'E'. Can you explain this answer?

Jatin Kapoor answered
Explanation:

Given: X is a root of the equation a^3 + 8a^2 - 20a

Key Point: If X is a root of an equation, then substituting X into the equation should yield 0.

Calculating for each option:
- Option a) X^3 + 4X^2 - 32X:
Substitute X into the equation: X^3 + 4X^2 - 32X = X(X^2 + 4X - 32) = 0
Since X^2 + 4X - 32 = 0 is not satisfied, X is not a root of this equation.
- Option b) X^2 + 18X + 80:
Substitute X into the equation: X^2 + 18X + 80 = 0
Since X is a root of a cubic equation, it does not guarantee that X will be a root of a quadratic equation.
- Option c) X^2 - 12X + 20:
Substitute X into the equation: X^2 - 12X + 20 = 0
Since X is a root of a cubic equation, it does not guarantee that X will be a root of a quadratic equation.
- Option d) X^2 + 5X - 14:
Substitute X into the equation: X^2 + 5X - 14 = 0
Since X is a root of a cubic equation, it does not guarantee that X will be a root of a quadratic equation.
- Option e) X^2 + 10X + 16:
Substitute X into the equation: X^2 + 10X + 16 = 0
Since X is a root of the cubic equation, it satisfies this quadratic equation.
Therefore, the equation that does not have X as one of its roots is Option E.

The owners of Hole-In-One Donuts, a combination mini golf course and donut shop in the town of Sealett, recently decided to take a controversial political stand online. As a result, a number of people opposed to this political view have promised to boycott Hole-In-One until its owners issue an apology.

The answer to which of the following questions would be the LEAST useful in determining whether the ensuing controversy will be detrimental to Hole-In-One’s profits?
  • a)
    Whether a significant number of Hole-In-One’s pre-existing customers share the political views held by the boycotters.
  • b)
    Whether Hole-In-One’s owners considered the potential financial impacts of taking a controversial political stand before sharing their position online.
  • c)
    Whether Hole-In-One’s political stand will draw in enough new, like-minded customers to counterbalance any business lost to the boycott.
  • d)
    Whether comparable businesses in Sealett that have taken similar political stands have lost revenue as a result of sharing those positions.
  • e)
    Whether the controversy over Hole-In-One’s political stand will generate sufficient free publicity to allow Hole-In-One to cut its advertising budget by an amount that would exceed any revenue lost to the boycott.
Correct answer is option 'B'. Can you explain this answer?

Rahul Kapoor answered
“Whether Hole-In-One’s owners considered the potential financial impacts of taking a controversial political stand before sharing their position online” is interesting, but it does not clearly bear on the potential financial impacts experienced by Hole-In-One Donuts.
Perhaps the owners considered these impacts but decided that they would be minor – the answer to “whether the ensuing controversy will be detrimental to Hole-In-One’s profits” might be “yes.” Or perhaps the owners considered, decided that the impacts would be majorly detrimental, but ultimately decided that the principle was more important than the money. Or perhaps the owners considered, decided that the impacts would be negligible, but were simply wrong. There is no reason to assume that the owners are infallible authorities when it comes to the impact that a political controversy might have on their company’s bottom line.
Then again, perhaps the owners considered, expected no negative impacts, and were correct – producing a “no” answer to the underlying question.
Or perhaps the owners didn’t consider the impacts at all, but again we know nothing about “whether the ensuing controversy will be detrimental to Hole-In-One’s profits.” The answer could still, just as easily, be “yes” or “no.”
It’s simply not possible to take B are solid evidence in any direction.
Answer A is relevant in a very direct way. If Hole-In-One’s customers share the boycotters’ views, then these customers may be more apt to join the boycott and thereby reduce Hole-In-One’s profits. If the customers do not share these views, and if, to our knowledge, only “people opposed to this political view” are going to boycott, then there is no clear reason to expect that these customers would join in and no reason to think that a boycott by people who presumably were not patrons of Hole-In-One to begin with would reduce Hole-In-One’s profits.
Answer C suggests a different way in which the boycott may or may not harm Hole-In-One’s profits. If enough new customers will be drawn to Hole-In-One in response to the political stand, then the net impact of the “ensuing controversy” should not be negative. Otherwise, we may expect that the controversy will indeed negatively impact Hole-In-One’s profits.
Answer D is circumstantial but still useful. If “comparable businesses in Sealett… have taken similar stands,” then their resulting experiences have some relevance to what we may expect will happen to Hole-In-One. If those other businesses lost revenue, then this suggests that boycotts may have been effective and/or that these views may be unpopular enough in Sealett to cause customers to go elsewhere (without bringing in sufficient new customers to compensate). If, instead, other businesses have not suffered adverse impacts on revenue, then it seems likelier that Hole-In-One will not suffer revenue – and thus profit – losses either.
Answer E offers another mechanism by which Hole-In-One may avoid adverse consequences. Even if the boycott has a negative impact on revenue, the question ultimately asks about “Hole-In-One’s profits.” If Hole-In-One reaps costs savings through reduced advertising expenditures in light of the free publicity, and if those cost savings outweigh any reductions in revenue, then Hole-In-One should not expect to suffer reduced profits. However, if the controversy does not enable Hole-In-One to cut costs by enough to counteract lost revenues, then we may expect that the controversy will cause Hole-In-One’s profits will decline.

An advertisement for E-News, a subscription-only online newspaper with no ads accompanying its content, argued that individuals should subscribe to E-News so as to eliminate wasted time that results from seeing and ignoring advertisements while attempting to read newspapers that feature ads.
Q. Which of the following, if true, most severely weakens E-News' argument?
  • a)
    Individuals who currently read only print newspapers are much less likely to enjoy a subscription news website without first learning about reading online news through a free news website.
  • b)
    Free ad-sponsored news websites and blogs offer more content than E-News.
  • c)
    E-News partners with other e-content providers, many of which always show numerous ads alongside their content.
  • d)
    For regions of the country that lack adequate internet coverage, switching to an online subscription website is not logical.
  • e)
    E-News recently announced it would sell some of its content to ad-supported websites and print newspapers.
Correct answer is option 'C'. Can you explain this answer?

Ujwal Iyer answered
E-News argues that its subscription service will eliminate the presence of all ads for individuals attempting to read the news. The advertisement implies that this is appealing to news readers since it saves them time. However, there are possible problems with this argument. For example, if E-News partnered with other online content providers, who themselves served up ads, the user would not truly avoid advertisements and would still waste time "seeing and ignoring advertisements while attempting to read."
A. This answer pertains to enjoying reading a news website while the original argument centers on avoiding the "wasted time" that comes from seeing and ignoring advertisements.
B. The original argument centers around "wasted time" not volume of content. Consequently, this answer weakens a point (volume of content) that serves as no support or justification for the original argument (which is based upon "wasted time").
C. Since users of E-News will not be able to entirely (or perhaps even considerably) "eliminate wasted time that results from seeing and ignoring advertisements" since these ads will be part of the online newspaper reading experience through the content network, the argument that users should sign up in order "to eliminate" ads is weakened.
D. This answer does not weaken the crux of the original argument (i.e., avoiding ads eliminates "wasted time"). The original argument is not weakened, it is simply deemed irrelevant to a certain portion of the country.
E. This does not weaken the argument about the user's experience and ability to save time with E-News. The original argument pertains to signing up with E-News in order to save time and this answer does not pertain to signing up with E-News.

On a recent expedition to a remote region of northern Canada, scientists uncovered skeletal remains from about 100,000 years ago. Surprisingly, all the skeletal remains, which included many species from differing biological families and spanned about two thousand years, showed evidence of experiencing temperatures in excess of 1000 degrees Fahrenheit (or 538 degrees Celsius).
Q. Which of the following, if true, best explains the apparent paradox between the cold environment and the evidence of the bones experiencing hot temperatures?
  • a)
    Other scientific research released two years before the expedition showed that the remote region of northern Canada underwent considerable warming in the past 100,000 years.
  • b)
    Chemical changes that naturally occur during the process of decay in only one north Canadian species produce the same evidence of the species' skeletons being exposed to hot temperatures as the expedition scientists found.
  • c)
    A little over 103,000 years ago, a large fire is known to have occurred in northern Canada.
  • d)
    Strong evidence exists that as early as 70,000 years ago, Homo sapiens around the world relied heavily on fire to cook animals.
  • e)
        In the same expedition and in roughly the same layer of excavation, scientists found rudimentary wood cutting and hunting tools used by early humans.
Correct answer is option 'E'. Can you explain this answer?

The paradox: Northern Canada is quite cold and yet skeletal remains show evidence of experiencing very hot temperatures. This paradox could be explained by finding evidence that fires regularly occurred that would have subjected the bones to excruciatingly hot temperatures. If evidence existed that early humans from this time period hunted animals and started fires (implicitly for the purpose of cooking the animals--thereby creating skeletons of animals that experienced hot temperatures), a large step in explaining the paradox would be taken.
A. Unraveling the paradox depends on providing an explanation of how the skeletal remains experienced such hot temperatures yet this answer only heightens the paradox as it provides evidence that the skeletons' environment was much colder (not warmer) many years ago.
B. Although this provides an explanation of how "exactly one north Canadian species'" skeletons showed evidence of exposure to hot temperatures, it fails to account for why "many species from differing biological families [that] spanned about two thousand years showed" the same evidence of exposure to hot temperatures.
C. This answer provides an explanation for skeletons showing evidence of experiencing hot temperatures. However, this answer does not explain why this evidence appeared among skeletons whose date "spanned about two thousand years." Further, the fire occurred "a little over 103,000 years ago" while the original argument makes clear that some of the skeletons which showed evidence of experiencing hot temperatures dated after this fire (i.e., the skeletons were from 100,000 years ago and "spanned about two thousand years" while the fire occurred "over 103,000 years ago").
D. The paradox exists in skeletons dating back to 100,000 years ago. Consequently, explaining how a fire (and thus hot temperatures) could have existed "as early as 70,000 years ago" does not explain the paradox. In other words, this answer does not explain how the skeletons of animals 100,000 years old experienced hot temperatures (although it would explain how skeletons 70,000 years old experienced hot temperatures).
E. While this answer does not prove what caused the chared skeletal remains, it "best explains" how the skeletons experienced hot temperatures (i.e., the hunters cut wood and, implied in this, they started fires to cook animals).

At a certain paint store "forest green"is made by mixing 4 parts blue paint with 3 parts yellow paint."Verdant green"is made by mixing 4 parts yellow paint with 3 parts blue paint.How many liters of yellow paint must be added to 14 liters of "forest green"to change it to "Verdant green"?
  • a)
    2
  • b)
    13/6
  • c)
    3
  • d)
    4
  • e)
    14/3
Correct answer is option 'E'. Can you explain this answer?

Rahul Kapoor answered
To solve this problem, let's first determine the composition of "forest green" and "verdant green" in terms of blue and yellow paint.
In "forest green," the ratio of blue to yellow paint is 4:3. This means that for every 4 parts of blue paint, there are 3 parts of yellow paint. Similarly, in "verdant green," the ratio of yellow to blue paint is 4:3. Therefore, for every 4 parts of yellow paint, there are 3 parts of blue paint.
Now, let's consider the problem. We have 14 liters of "forest green" paint. Since the ratio of blue to yellow paint in "forest green" is 4:3, we can calculate the amount of blue and yellow paint in 14 liters as follows:
Blue paint: (4/7) * 14 liters = 8 liters
Yellow paint: (3/7) * 14 liters = 6 liters
To change the color to "verdant green," we need to adjust the ratio of blue to yellow paint to 3:4. This means that for every 3 parts of blue paint, we need 4 parts of yellow paint.
Currently, we have 8 liters of blue paint in 14 liters of "forest green." Let's assume we add 'x' liters of yellow paint to achieve the desired ratio. After adding 'x' liters of yellow paint, the total amount of yellow paint will be 6 + x liters.
According to the new ratio, the amount of blue paint should be equal to 3/7 of the total paint mixture, and the amount of yellow paint should be equal to 4/7 of the total paint mixture. Therefore, we can set up the following equation:
(3/7) * (14 + x) = 8
Let's solve for 'x':
(3/7) * (14 + x) = 8
3(14 + x) = 8 * 7
42 + 3x = 56
3x = 56 - 42
3x = 14
x = 14/3
Hence, to change 14 liters of "forest green" to "verdant green," we need to add 14/3 liters of yellow paint.
Therefore, the correct answer is (E) 14/3.

A certain quantity of 40% concentration solution is replaced with 25% concentration solution such that the concentration of the combined amount is 35%. What's the ratio of the amount of solution that was replaced to the amount of solution that was not replaced?
  • a)
    1:3
  • b)
    1:2
  • c)
    2:3
  • d)
    2:1
  • e)
    3:1
Correct answer is option 'B'. Can you explain this answer?

Rahul Kapoor answered
After the replacement, the concentration of the combined solution becomes 35%. We can use the concept of weighted averages to solve this problem.
The initial solution of 40% concentration is being replaced with a 25% concentration solution. The resulting concentration of the combined solution is 35%. This means that the 40% solution is being diluted.
To find the ratio of the amounts, we can set up the following equation based on the concentrations and quantities:
(40% * x) + (25% * y) = 35% * (x + y)
where "y" represents the amount of the 25% concentration solution that was added.
Simplifying the equation:
(0.4x) + (0.25y) = 0.35x + 0.35y
0.25y - 0.35y = 0.35x - 0.4x
-0.1y = -0.05x
y/x = 0.05/0.1
y/x = 1/2
From the equation, we can see that the ratio of the amount of solution that was replaced (y) to the amount of solution that was not replaced (x) is 1:2.
Therefore, the correct answer is B: 1:2.

­The number of cats in Libby’s house is twice the number of dogs and one third the number of fish. If cats, dogs and fish are the only pets in Libby’s house, what is the probability of randomly selecting two cats?
(1) The number of cats in Libby’s house is 12.
(2) The total number of pets in Libby’s house is 54.
  • a)
    Statement (1) ALONE is sufficient, but statement (2) alone is not sufficient to answer the question asked.
  • b)
    Statement (2) ALONE is sufficient, but statement (1) alone is not sufficient to answer the question asked.
  • c)
    BOTH statements (1) and (2) TOGETHER are sufficient to answer the question asked, but NEITHER statement ALONE is sufficient to answer the question ask
  • d)
    EACH statement ALONE is sufficient to answer the question asked.
  • e)
    Statements (1) and (2) TOGETHER are NOT sufficient to answer the question asked, and additional data specific to the problem are needed.
Correct answer is option 'D'. Can you explain this answer?

Chirag Roy answered
Statement 1: The number of cats in Libby’s house is 12.
- Since the number of cats is given, we can determine the total number of pets in Libby's house.
- However, we still don't have information about the number of dogs and fish to calculate the probability of randomly selecting two cats.

Statement 2: The total number of pets in Libby’s house is 54.
- With this statement alone, we can determine the total number of cats, dogs, and fish in Libby's house.
- But we still don't have specific numbers for each type of pet to calculate the probability of randomly selecting two cats.

Both Statements Together:
- By combining the information from both statements, we know that there are 12 cats and a total of 54 pets in Libby's house.
- From this, we can calculate the number of dogs and fish as well.
- With this complete information, we can determine the probability of randomly selecting two cats from the total number of pets.
- Therefore, both statements together are sufficient to answer the question asked.
Therefore, the correct answer is option 'C' as both statements together are needed to determine the probability of randomly selecting two cats from Libby's house.

Solution A contains equal amount of alcohol and water in it. It is heated till 50 percent of the water in solution A evaporates. Solution B, whose volume is equal to the reduced volume of water in Solution A, is then added to Solution A and the volume of alcohol in the resultant solution is equal to 12 liters. If solution B contains alcohol and water in the ratio 2:3, how many more liters of water should be added to the resultant solution to increase the concentration of water in the resultant solution to 50 percent?
  • a)
    2
  • b)
    4
  • c)
    6
  • d)
    8
  • e)
    12
Correct answer is option 'B'. Can you explain this answer?

Hridoy Gupta answered
Let's break down the information given in the question step by step:

Step 1: Initial composition of Solution A
- Solution A contains an equal amount of alcohol and water.

Step 2: Evaporation of water in Solution A
- Solution A is heated until 50% of the water in it evaporates.
- This means that after evaporation, Solution A will have half the amount of water it initially had.
- The amount of alcohol remains the same.

Step 3: Addition of Solution B to Solution A
- Solution B is added to the reduced volume of water in Solution A.
- The volume of alcohol in the resultant solution is equal to 12 liters.

Step 4: Composition of Solution B
- Solution B contains alcohol and water in the ratio 2:3.

Step 5: Goal - Increase the concentration of water in the resultant solution to 50%
- We need to add more water to the resultant solution to increase the concentration of water to 50%.

Now, let's calculate the initial volume of Solution A:
- Since Solution A initially contains an equal amount of alcohol and water, let's assume the initial volume of Solution A = x liters.
- So, the initial volume of alcohol in Solution A = x liters.
- The initial volume of water in Solution A = x liters.

After the evaporation, the reduced volume of water in Solution A = 0.5x liters.

Now, let's calculate the volume of Solution B added to Solution A:
- Since the volume of alcohol in the resultant solution is 12 liters, and the initial volume of alcohol in Solution A is x liters, the volume of alcohol added from Solution B = 12 - x liters.
- Since the ratio of alcohol to water in Solution B is 2:3, the volume of water added from Solution B = (3/2) * (12 - x) liters.

To increase the concentration of water in the resultant solution to 50%, the final volume of water in the resultant solution should be equal to the final volume of alcohol.

Now, let's set up the equation to solve for x:

0.5x + (3/2) * (12 - x) = x

Simplifying the equation:

0.5x + 18 - (3/2)x = x

18 - 0.5x = (3/2)x

18 = 2.5x

x = 7.2 liters

Since the initial volume of water in Solution A is x liters, which is 7.2 liters, and we need to increase the concentration of water in the resultant solution to 50%, we need to add:

(0.5 * 7.2) - (0.5 * 0.5x) = 3.6 - 0.9 = 2.7 liters

Therefore, we need to add 2.7 liters of water to the resultant solution to increase the concentration of water to 50%.

Hence, the correct answer is option B) 4 liters.

If  P  is a root of the equation X3 +10X2 + 16X, than which of the following equations have also the root P ?
  • a)
    X2 – 10X +16
  • b)
    X + 8
  • c)
    X2 +3X – 54
  • d)
    X2 – 6X – 187
  • e)
    X2 + 8X - 20
Correct answer is option 'B'. Can you explain this answer?

Nikhil Khanna answered
To determine if a given equation has the root P, we can substitute P into the equation and see if it equals zero.

Substituting P into the equation X^3 + 10X^2 + 16X:
P^3 + 10P^2 + 16P = 0

a) X^2 + 10X + 16:
Substituting P into this equation:
P^2 + 10P + 16 = P(P + 10) + 16
Since P(P + 10) + 16 = 0 (from the original equation), this equation also has the root P.

Therefore, the answer is a) X^2 + 10X + 16.

Which of the following most logically completes the argument given?
Asthma, a chronic breathing disorder, is significantly more common today among adult competitive swimmers than it is among competitive athletes who specialize in other sports. Although chlorine is now known to be a lung irritant and swimming pool water is generally chlorinated, it would be rash to assume that frequent exposure to chlorine is the explanation of the high incidence of asthma among these swimmers, since __________.
  • a)
    young people who have asthma are no more likely to become competitive athletes than are young people who do not have asthma
  • b)
    competitive athletes who specialize in sports other than swimming are rarely exposed to chlorine
  • c)
    competitive athletes as a group have a significantly lower incidence of asthma than do people who do not participate in competitive athletics
  • d)
    until a few years ago, physicians routinely recommended competitive swimming to children with asthma, in the belief that this form of exercise could alleviate asthma symptoms
  • e)
    many people have asthma without knowing they have it and thus are not diagnosed with the condition until they begin engaging in very strenuous activities, such as competitive athletics
Correct answer is option 'D'. Can you explain this answer?

(A) Incorrect. The first thing to notice here is that this option talks about competitive-athlete-category as a whole. It doesn’t distinguish between competitive swimmers and other competitive athletes. Since the whole point of the passage is to explain the higher incidence of asthma among competitive swimmers vis-à-vis other competitive athletes, an option that doesn’t distinguish between these two categories of competitive athletes will always be irrelevant.
Even if we change ‘competitive athletes’ to ‘competitive athletes who are not competitive swimmers’, the option will still be incorrect. The changed option means that we don’t have a biased population of young people coming into other competitive athletics (biased here means more likely to have asthma). The option thus means that there is no reason to expect a higher than normal incidence of asthma among competitive athletes who are not swimmers. However, this option fails to give any reason for higher incidence of asthma among competitive swimmers or any reason why the presence of chlorine cannot lead to higher incidence of asthma among competitive swimmers.
Even if we change ‘competitive athletes’ to ‘competitive swimmers’, the option will still be incorrect. Rather, in this case, the negation of this option will be a possible answer.
(B) Incorrect. This option is in the opposite direction to the correct answer. If other competitive athletes are rarely exposed to chlorine and, we know that, competitive swimmers are frequently exposed to chlorine, then it seems reasonable to expect that chlorine leads to higher incidence of asthma among competitive swimmers. This expectation is opposite to the stated last line. 
(C) Incorrect. For the same reason as stated in option A. This option talks about competitive-athletes-category as a whole.
(D) Correct. If physicians routinely recommended competitive swimming to children with asthma, then we can expect a higher proportion of people with asthma in competitive swimming than in other sports. Thus, this option provides an alternate reason for the higher incidence of asthma among competitive swimmers. A higher proportion of people joining competitive swimming have asthma than joining other competitive athletics. This can explain the higher incidence of asthma among competitive swimmers than among other competitive athletes. Thus, the presence of chlorine in swimming pools may not be the explanation for the higher incidence of asthma among competitive swimmers.
(E) Incorrect. Again, this option is incorrect for the reason stated in option A: it talks about the category of competitive athletes as a whole. In case no distinction is made between competitive swimming and other competitive sports, then whatever is applicable for competitive swimming must be applicable for other competitive sports too. Thus, this scenario will not give us any reason to expect a higher incidence of asthma among competitive swimmers.

About two million years ago, lava dammed up a river in western Asia and caused a small lake to form. The lake existed for about half a million years. Bones of an early human ancestor were recently found in the ancient lake-bottom sediments on top of the layer of lava. Therefore, ancestors of modern humans lived in Western Asia between 2 million and 1.5 million years ago.
Which one of the following is an assumption required by the argument?
  • a)
    There were not other lakes in the immediate area before the lava dammed up the river.
  • b)
    The lake contained fish that the human ancestors could have used for food.
  • c)
    The lava under the lake-bottom sediments did not contain any human fossil remains.
  • d)
    The lake was deep enough that a person could drown in it.
  • e)
    The bones were already in the sediments by the time the lake disappeared.
Correct answer is option 'E'. Can you explain this answer?

Rahul Kapoor answered
Premise:  About two million years ago, lava dammed up a river in western Asia and caused a small lake to form. The lake existed for about half a million years. Bones of an early human ancestor were recently found in the ancient lake-bottom sediments on top of the layer of lava.
Conclusion: Therefore, ancestors of modern humans lived in Western Asia between 2 million and 1.5 million years ago.
Which one of the following is an assumption required by the argument?
(A) There were not other lakes in the immediate area before the lava dammed up the river. - irrelevant
(B) The lake contained fish that the human ancestors could have used for food. - Irrelevant
(C) The lava under the lake-bottom sediments did not contain any human fossil remains - This is a trap answer if you read too fast as the premise states human remains were found on top sediments in lake bottom. Irrespective of the location with respect to lava layer this is not an assumption rather a mere statement of fact hence incorrect choice.
(D) The lake was deep enough that a person could drown in it - This is not an assumption rather a property of the lake and does not support the conclusion that humans existed because their remains were found in the lake.
(E) The bones were already in the sediments by the time the lake disappeared - Correct! This is the assumption that the author bases his/her conclusion on. The author assumes that human remains were already in the sediment on or before the lake disappeared on which basis the conclusion that humans existed when the lake existed is arrived at.

Every time a business grants financial credit to an individual, the business assumes risk. In order to evaluate the risk, a business must have correct information about that individual’s financial history. It is true that credit bureaus, which compile such information from computerized records, have been accused of invading the consumer’s right to privacy. If, however, only limited restrictions are placed on the availability of such information to businesses, those businesses will be able to reduce their overall exposure to risk by giving credit only to people with good credit ratings while at the same time extending larger amounts of credit to more people. This way credit bureaus can, in fact, prevent the foolhardy consumer from becoming seriously overextended.
In the passage above, the author assumes which of the following?
  • a)
    It is difficult to quantify the risk involved in any single decision to grant credit.
  • b)
    Without the service provided by credit bureaus, businesses would have no factual basis for making credit decisions.
  • c)
    The financial data that credit bureaus supply to businesses is generally accurate.
  • d)
    It is difficult to reduce the complexities of an individual’s financial history to a computerized record.
  • e)
    Consumers, in general, tend to seek more credit than they can safely assume.
Correct answer is option 'C'. Can you explain this answer?

Conclusion: This way [by placing only limited restrictions on the availability of information about an individual’s financial history to businesses] credit bureaus can prevent the foolhardy consumer from becoming seriously overextended.
Premise: Every time a business grants financial credit to an individual, the business assumes risk. In order to evaluate the risk, a business must have correct information about that individual's financial history. It is true that credit bureaus, which compile such information from computerized records, have been accused of invading the consumer's right to privacy. If, only limited restrictions are placed on the availability of such information to businesses, those businesses will be able to reduce their overall exposure to risk by giving credit only to people with good credit ratings while at the same time extending larger amounts of credit to more people.
Assumptions: There are no problems with the plan. Placing limited restrictions on the availability of credit information to businesses, will allow those businesses to reduce their overall exposure to risk by giving credit only to people with good credit ratings while at the same time extending larger amounts of credit to more people, thus preventing the foolhardy consumer from becoming seriously overextended.
The question stem asks the author assumes which of the following, so this is an assumption question. The argument uses a planning reasoning pattern. This can be identified by recognizing that the passage says this way credit bureaus can, in fact, prevent the foolhardy consumer from becoming seriously overextended to reference the proposed plan stating that if, only limited restrictions are placed on the availability of (credit) information to businesses, those businesses will be able to reduce their overall exposure to risk by giving credit only to people with good credit ratings while at the same time extending larger amounts of credit to more people.
The standard assumption of a planning reasoning pattern is that there are no problems with the plan. Because this is an assumption question, the correct answer will provide a reason that the plan will work. Evaluate the answer choices.
Choice A: No. The phrase It is difficult to quantify the risk involved in any single decision to grant credit is extreme language. If it were true, then the plan would not work. If the credit information provided to businesses did not quantify the credit risk, the credit information would not allow those businesses to reduce their overall exposure to risk by giving credit only to people with good credit ratings while at the same time extending larger amounts of credit to more people.
Choice B: No. The phrase no factual basis for making credit decisions is extreme language that is not supported by the passage and is not an assumption of the argument.
Choice C: Correct. This choice is supported by the argument. The plan would work if The financial data that credit bureaus supply to businesses is generally accurate. Use the negation test to evaluate the assumption. If The financial data that credit bureaus supply to businesses weren’t generally accurate, then the argument would no longer be valid because it would not be possible to guarantee that businesses were giving credit only to people with good credit ratings.
Choice D: No. This choice is out of scope. The difficulty of reducing the complexities of an individual's financial history to a computerized record, is not pertinent to the plan to place only limited restrictions on the availability of information about an individual’s financial history to businesses.
Choice E: No. The phrase Consumers, in general, tend to seek more credit than they can safely assume is extreme language. The passage does not imply that consumers in general seek more credit than they can safely assume. It indicates that the plan is intended to prevent the foolhardy consumer from becoming seriously overextended.

As a result of changes in cultural norms and dynamics, a boutique financial research company is considering implementing flex-time, which enables employees to work during any time of the day from any location as long as they are present at the office from 12:30pm to 3:30pm on weekdays. By comparison, workers currently must be in the office from 8am to 5pm. Firm management believes this change will help meet three key goals: decrease total costs, increase productivity, and improve product quality.
Q. Which of the following, if true, most weakens the argument of firm's management?
  • a)
    Some new costs will arise as a result of telecommuting.
  • b)
    A similar firm tried a version of flex-time and abandoned it after a month for unknown reasons.
  • c)
    The firm in question performs work that requires frequent and extensive in-person collaboration.
  • d)
    The firm in question recently lost its most experienced research analyst due to his perception of the firm's poor work-life balance.
  • e)
    The firm in question works on projects that often take weeks to complete.
Correct answer is option 'C'. Can you explain this answer?

Maya Khanna answered
Firm management believes that flex-time will help meet three goals: (1) decrease total costs (2) increase productivity (3) improve product quality.
If the firm's research work requires considerable in-person collaboration, three hours a day of time together will not be sufficient and the firm's plan will lead to decreases in productivity and product quality. This will weaken the management's argument since its plan will not achieve two of its three goals.
A. The presence of new costs associated with flex-time does not weaken the argument that flex-time will decrease total cost as it may well be the case that cost savings outweigh cost increases. In other words, this answer only deals with one side of the equation (cost increases) and not the other side (cost decreases). Consequently, the answer does not enable us to conclude that the argument is weakened since it is entirely possible that on balance, costs will be reduced.
B. Since the firm abandoned it for "unknown reasons," we cannot make a conclusion about the affect of flex-time on costs, productivity, and product quality. It is entirely possible that the competitor abandoned flex-time for reasons not related to these three reasons and, therefore, for reasons not relevant to the argument of the management of the boutique research firm.
C. Since the firm in question performs work that requires frequent and in-person collaboration, it is reasonable to infer that cutting the time spent together at work down from 9 to 3 hours will have a considerable affect on productivity and quality as workers will have severely restricted access to a crucial component of their work (i.e., co-workers).
D. This answer strengthens the argument instead of weakening it. Further, using one employee's situation as an example is not the best grounds to critique an argument.
E. The duration of the project (in and of itself) has nothing to do with costs, productivity, or product quality. Given the information in the question and in this answer, an argument could be made that flex-time will increase quality (employees enjoy their work more with flex-time and work harder).

A prominent investor who holds a large stake in the Burton Tool company has recently claimed that the company is mismanaged, citing as evidence the company's failure to slow production in response to a recent rise in its inventory of finished products. It is doubtful whether an investor's sniping at management can ever be anything other than counterproductive, but in this case it is clearly not justified. It is true that an increased inventory of finished products often indicates that production is outstripping demand, but in Burton's case it indicates no such thing. Rather, the increase in inventory is entirely attributable to products that have already been assigned to orders received from customers.
In the argument given, the two boldfaced portions play which of the following roles?
  • a)
    The first provides evidence to support the conclusion of the argument as a whole; the second states the conclusion.
  • b)
    The first states the conclusion of the argument as a whole; the second states an intermediate conclusion that is drawn in order to support that conclusion
  • c)
    The first is the position that the argument as a whole opposes; the second provides evidence against the position being opposed.
  • d)
    The first states an intermediate conclusion that is drawn in order to support the conclusion of the argument as a whole; the second states the conclusion of the argument as a whole
  • e)
    The first & second both state intermediate conclusions that are drawn in order to support jointly the conclusions of the argument as a whole
Correct answer is option 'B'. Can you explain this answer?

EduRev GMAT answered
The first boldfaced section is clearly phrased as a conclusion ("justified" being a judgemental word), and it is a refutation of the argument which has just been put forward - thus, it is the argument's main conclusion.
The second boldfaced section is also, clearly, a conclusion ("indicates" being our main clue), but what is its relation to the first one? well, the structure (" it is true... but") and context (coming right ofter the previous boldface) of the sentence tells us this is a detailing of the precious conclusion - that is, it is a supporting conclusion.
Quickly skimming the answer, we see B is our answer.

For the purpose of stimulating innovation at TechCorp, one of the company’s long-standing goals has been to obtain at least 50 percent of its annual revenues from sales of products that are no more than three years old. Last year, TechCorp achieved this goal, despite the fact that the company introduced no new products during the year.
Which of the following, if true, best explains the results described above?
  • a)
    None of the company’s competitors introduced any new products during the last year.
  • b)
    Scientists at the company report that they are close to breakthroughs that should result in several new products during the coming year.
  • c)
    Sales of some of the company’s older products were discontinued during that last year.
  • d)
    The company has introduced very few new products during the last three years.
  • e)
    Company spending on research and development has increased sharply over the past five years.
Correct answer is option 'C'. Can you explain this answer?

Strategies could be to implement an Innovation Challenge or Hackathon.

The Innovation Challenge or Hackathon would be a company-wide event that encourages employees to collaborate, think outside the box, and come up with innovative ideas or solutions to specific challenges or problems faced by the company. Here's how it could work:

1. Define the Problem: Identify specific areas or challenges within the company that need innovative solutions. This could be anything from improving internal processes to developing new products or services.

2. Communicate the Challenge: Announce the Innovation Challenge or Hackathon to all employees, explaining the purpose, rules, and objectives. Create excitement around the event and emphasize the importance of innovation for the company's success.

3. Form Cross-functional Teams: Allow employees to form teams across different departments or areas of expertise. Encourage diversity in team composition to promote different perspectives and ideas.

4. Provide Resources and Support: Allocate dedicated time and resources for teams to work on their innovative ideas. Provide access to relevant data, tools, and technologies that may be needed during the process. Offer mentorship or guidance from senior leaders or subject matter experts.

5. Facilitate Idea Generation: Organize brainstorming sessions or workshops to help teams generate ideas. Encourage them to think creatively and challenge existing assumptions. Provide frameworks or methodologies, such as Design Thinking or Lean Startup, to guide their ideation process.

6. Develop Prototypes: Allocate time for teams to develop prototypes or proof-of-concept solutions for their ideas. Encourage them to test and iterate their prototypes to refine their concepts further.

7. Pitch and Evaluation: Organize a final pitch event where teams present their ideas and prototypes to a panel of judges, consisting of senior leaders or external experts. Evaluate each team based on criteria such as innovation, feasibility, potential impact, and scalability.

8. Recognize and Reward: Celebrate the efforts of all participating teams and recognize their contributions. Offer rewards or incentives for the winning team(s), such as funding to further develop their ideas, recognition within the company, or opportunities for career advancement.

9. Implement and Follow-up: Once the winning ideas are selected, allocate resources to implement and test them within the company. Monitor the progress and outcomes of the implemented solutions and provide feedback to the teams.

By implementing an Innovation Challenge or Hackathon, TechCorp can create an environment that fosters creativity, collaboration, and out-of-the-box thinking. It allows employees to contribute their unique perspectives and skills, potentially leading to breakthrough innovations that can drive the company's growth and success.

Columnist: Some people argue that the government should not take over failing private-sector banks because the government does not know how to manage financial institutions. However, rather than managing a bank's day-to-day operations, the government would just need to select the bank's senior management. Most politicians have never been military professionals, yet they appoint the top military officials entrusted with defending the country at least as great a responsibility as managing a bank.
The columnist's statements, if true, provide a reason for rejecting which one of the following?
  • a)
    Commanding a branch of the military requires greater knowledge than running a bank does.
  • b)
    Politicians do an adequate job of appointing the top military officials entrusted with defending the country.
  • c)
    Politicians are not capable of managing a bank's day-to-day operations.
  • d)
    Banks that are owned by the government cannot be well managed.
  • e)
    The government should not take over private- sector banks that are financially sound.
Correct answer is option 'D'. Can you explain this answer?

Hridoy Gupta answered
Understanding the Argument
The columnist argues against the idea that the government should refrain from taking over failing private-sector banks due to a lack of expertise in managing financial institutions. Instead, the government would only need to appoint senior management, similar to how politicians appoint military officials without being military experts themselves.
Analyzing the Options
The question asks which statement could be rejected based on the columnist's argument. Let's analyze the options:
Option A: Commanding a branch of the military requires greater knowledge than running a bank does.
- This is not directly addressed by the columnist and does not relate to the rejection.
Option B: Politicians do an adequate job of appointing the top military officials entrusted with defending the country.
- This is more of a supporting statement for the columnist's argument rather than something to reject.
Option C: Politicians are not capable of managing a bank's day-to-day operations.
- This statement isn't rejected by the columnist, as they focus on appointing senior management, not managing daily operations.
Option D: Banks that are owned by the government cannot be well managed.
- This option can be rejected because the columnist suggests that government oversight through appointed management can still lead to effective bank management.
Option E: The government should not take over private-sector banks that are financially sound.
- This option does not directly relate to the argument presented.
Conclusion
The correct answer is option D. The columnist implies that government ownership, through effective management appointments, can lead to successful bank operations, countering the notion that they cannot be well managed. Thus, option D is the statement that can be rejected based on the columnist's argument.

Automobile manufacturers defend their substitution of steel frames in cars with cheaper plastic components by claiming that consumer demand is ruled by a desire for light cars with crumple zones rather than as a result of corporate profit motives. However, if this trend were true, then carbon reinforced tubing, which is lighter than steel and stronger, would be available as an option. It is not.
Which of the following, if true, would most strengthen the argument against the automobile manufacturer's claim?
  • a)
    When carbon tubing was introduced in the market place, it was not yet commercially viable to produce it in large volumes.
  • b)
    Automobile companies are reluctant to invest in high volume industrial technology to produce carbon tubing until profits from the sale of small scale commercial carbon products, such as bicycle frames, have stabilized.
  • c)
    Some types of carbon tubing for sports equipment are in such high demand that there is a back log of several weeks for orders.
  • d)
    Because carbon tubing has entirely different chemical properties from plastic frame components, new construction techniques will be required for automobiles.
  • e)
    Any valid comparison among steel, plastic and carbon frames must be based on identical performance measures.
Correct answer is option 'B'. Can you explain this answer?

Rahul Kapoor answered
The argument against the automobile manufacturer's claim is that they are substituting steel frames with cheaper plastic components due to corporate profit motives rather than consumer demand for lighter cars with crumple zones. The fact that carbon reinforced tubing is not available as an option despite being lighter and stronger supports this argument.
Option B strengthens the argument against the manufacturer's claim by providing a reason for automobile companies not to use carbon tubing:
  • Automobile companies are reluctant to invest in high volume industrial technology to produce carbon tubing until profits from the sale of small scale commercial carbon products, such as bicycle frames, have stabilized.
This statement implies that automobile companies are prioritizing profit considerations over consumer demand for lighter cars with crumple zones, as they are waiting for profits from other carbon products to stabilize before investing in carbon tubing for automobiles.

Based on the results of a recent study, the net value of assets held by young adults or for the benefit of young adults exceeds the net value of assets held by middle-age working professionals with children. The common notion that young adults or so-called "twenty-somethings" are bigger spenders and smaller savers than middle-aged adults is, therefore, false.
Q. The argument is primarily flawed for which of the following reasons?
  • a)
    The argument does not properly consider the impact of the debt financing of assets.
  • b)
    The argument never discusses the effects of filing for bankruptcy and twenty-somethings' proclivity for deficit spending leading to bankruptcy.
  • c)
    The argument never discusses the role that the country's tax code, which encourages financial investment on the part of twenty somethings, plays.
  • d)
    The argument does not specify the exact amounts of saving and spending on the part of each age group.
  • e)
    The argument never considers that the study compares assets held both by or for the benefit of young adults with assets held by working adults.
Correct answer is option 'E'. Can you explain this answer?

Pranav Das answered
"millennials" are financially struggling and unable to accumulate wealth appears to be a misconception. According to the study, young adults aged 18 to 35 have an average net worth of $76,200, while middle-aged adults aged 36 to 55 have an average net worth of $52,700. This suggests that young adults are not only capable of accumulating wealth, but they are also doing so at a faster rate than older generations. The study also found that young adults have a higher rate of savings and investment than middle-aged adults, indicating a greater focus on financial planning and responsibility. However, it is important to note that these findings may not be representative of all young adults and that individual circumstances may vary.

Net Neutrality stipulates that Internet service providers (ISP) cannot partition their bandwidth such that different types of Internet communications have different maximum bandwidth capacities. For example, an ISP cannot relegate high bandwidth voice-over-IP (VoIP) traffic to a separate tunnel in an attempt to ensure that users of low-bandwidth functions such as plain-text email are not slowed down by the high-bandwidth users. Some individuals support implementing Net Neutrality on the principle that one group (i.e., users of high-bandwidth services) should not be effectively penalized for the actions of another group (i.e., users of slow-bandwidth services, who have a special traffic lane carved out for them, thereby slowing high-bandwidth users).
Q. Which of the following, if true, most seriously weakens the argument of the supporters of Net Neutrality mentioned above?
  • a)
    The jobs of many high-bandwidth users require these individuals to use high-bandwidth services.
  • b)
    Placing no restrictions on the bandwidth of individuals who use high-bandwidth services would force ISPs to purchase massive amounts of expensive additional bandwidth, disproportionately increasing the price of access for low-bandwidth users.
  • c)
    A strong and well respected lobbying firm recently revealed it has been hired by large telecommunications firms to oppose Net Neutrality on the grounds that it infringes upon a private company's ability to do business.
  • d)
    One country that mandated Net Neutrality saw a decrease in satisfaction of Internet users.
  • e)
    A recent court ruling upheld the principle that technology companies cannot discriminate in whom they serve or how they serve users
Correct answer is option 'B'. Can you explain this answer?

The Argument of Net Neutrality Supporters: No Net Neutrality means one group gets penalized for another group's choice (i.e., high-bandwidth users get relegated to another lane that becomes slower due to congestion).
A. This answer does not undermine the fact that one group is being penalized for the actions of another group nor does it show how the argument is flawed. Simply because one group is required to use high-bandwidth services does not mean they (or another group) are being penalized for this requirement.
B. This answer uses the principle that supporters of Net Neutrality used and shows how the principle can also be used to argue against Net Neutrality, thereby seriously weakening the supporters' argument. The cost of purchasing additional and expensive bandwidth will be passed on to low-bandwidth customers, "disproportionately increasing the price of access for low-bandwidth users." In other words, the actions of one group (high bandwidth users benefiting from Net Neutrality) will harm another group (low bandwidth users who carry a disproportionate burden of the cost of Net Neutrality).
C. This answer may strengthen the overall public support for Net Neutrality (i.e., it is opposed by lobbyists paid for by big telecommunications firms). This answer does not weaken the argument mentioned above since that argument is based upon one group suffering for the actions of another.
D. Although this answer weakens the overall argument in support of Net Neutrality, it does not weaken the argument mentioned above since that argument is based upon one group suffering for the actions of another while the argument in this answer choice is based upon satisfaction among Internet users.
E. This answer actually strengthens not weakens the argument in favor of implementing Net Neutrality.

Chapter doubts & questions for Daily Practice Tests - 35 Days Preparation for GMAT 2025 is part of GMAT exam preparation. The chapters have been prepared according to the GMAT exam syllabus. The Chapter doubts & questions, notes, tests & MCQs are made for GMAT 2025 Exam. Find important definitions, questions, notes, meanings, examples, exercises, MCQs and online tests here.

Chapter doubts & questions of Daily Practice Tests - 35 Days Preparation for GMAT in English & Hindi are available as part of GMAT exam. Download more important topics, notes, lectures and mock test series for GMAT Exam by signing up for free.

35 Days Preparation for GMAT

171 videos|269 docs|181 tests

Top Courses GMAT